The Case for Mrs. Cruz
Name that Arthritis
Walgreens
Draw it to know it
Spiral
100

These specific alleles are linked to rheumatoid arthritis, and these alleles share a common sequence of amino acids in a polymorphic region of the β chain, which is designated the shared epitope 

HLA-DRB1

100

A 62-year-old woman comes to the office because of pain in her hands for three years. She describes the pain as progressive - coming on slowly and worsening over the course of the three years. She says that her aunt and wheelchair-bound grandmother had problems with their hands as well. A hand X-ray is obtained which shows soft tissue swelling and marked juxta-articular osteopenia in her metacarpophalangeal and proximal interphalangeal joints, and minor bony erosions. Which of the following will most likely be present on her plain film ten years from now?

 

  1. No further progression
  2. Pencil-in-cup deformity
  3. Subchondral sclerosis, subchondral cysts, osteophytes, joint space narrowing
  4. Subluxations (e.g. boutonniere and swan-neck deformities), joint ankylosis, z-thumb deformity
  5. Subperiosteal bone resorption

D. Explanation: Based on the X-ray findings and family history, this lady is likely now manifesting rheumatoid arthritis (RA) in her hands. Classic early findings in RA include soft tissue swelling, osteopenia, erosions, and joint-space narrowing. Later changes include subluxation causing ulnar deviation, z-thumb, boutonniere and swan neck deformities, and ankylosis.

100

These are the things that DMARDs affect/inhibit in order to be effective at managing RA

T-cells (Abatacept) 

B-cells (Rituximab)

TNF, IL-1, IL-6


100

When the APC interacts with the citrullinated peptide & mounts an immune response (i.e. it does not recognize it as self) it goes through THIS process (draw it) in THIS place (specific location) to activate the adaptive immune response 

Schematic of T-cell activation (including B-cell activation is optional)

100

Describe the morphological changes occurring in the synovial space (be sure to include why & reversibility if applicable) (Unit 3 spiral) 

Some examples: 

Synovial cells - hyperplasia (reversible?)

Cartilage - necrosis (irreversible) 


200

A 45-year-old woman comes to the office because of pain, stiffness, and swelling of the small joints of the hands and feet for 3 months. She also has increasing fatigue that has caused her to miss work at least 1 day per week. She has no other medical problems. Physical examination shows the vital signs are within the normal range. A photograph of one of her hands is shown. Complete blood count, serum chemistries, and urinalysis are all normal. Erythrocyte sedimentation rate is elevated at 44 mm/h. Which of the following is the most likely diagnosis?

  1. Osteoarthritis
  2. Psoriatic Arthritis
  3. Rheumatoid Arthritis
  4. SLE
  5. Gout

 

C. Explanation: This patient has symptoms and signs consistent with rheumatoid arthritis. Different joints are variably affected by different disorders. Rheumatoid arthritis and osteoarthritis can both involve the proximal interphalangeal joints of the hands, but metacarpophalangeal joint involvement occurs in rheumatoid arthritis but not typically in osteoarthritis.  This patient has erythema and swelling of the metacarpophalangeal joints and loss of function leading to absenteeism from work; these findings are most consistent with rheumatoid arthritis. Women are more commonly affected and HLA-DR4 is associated. High rheumatoid factor (RF) is associated with severe disease, although 3% of the healthy population has RF. Synovial analysis will show findings consistent with an inflammatory process

200

A 67-year-old male with no significant medical history comes to your office complaining of pain and stiffness in his hands upon wakening in the morning. You perform an x-ray (Figure A). What is the most likely diagnosis?

Figure A: <img data-fr-image-pasted="true" data-imagetype="AttachmentByCid" data-custom="AQMkADk0OWNlMzc4LTBmMDgtNDQCLTkxZjUtMDNjOGU0MTViNThhAEYAAANHwy14K%2FLTR6IO4dn8sL4XBwB9w94VIjJYT6J5Nb5ErSuIAAACAQwAAAB9w94VIjJYT6J5Nb5ErSuIAAAA%2FNVkgQAAAAESABAA2vT9lnFYsUmHhDfBATFu6w%3D%3D" src="data:image/png;base64,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

Answer: B

This radiograph demonstrates complete loss of the articular cartilage at all four distal interphalangeal (DIP) joints, large osteophytes, and ankylosis of the DIP joint of the middle finger, making OA (answer B) the most likely diagnosis.

200

Identify key drug interactions for the medications Mrs. Cruz is taking 

Some examples: 

Corticosteroids (Systemic) may enhance the adverse/toxic effect of Nonsteroidal Anti-Inflammatory Agents (Nonselective).

Vaccines (Live): Methotrexate may enhance the adverse/toxic effect of Vaccines (Live). Methotrexate may diminish the therapeutic effect of Vaccines (Live). Management: Low-dose methotrexate (0.4 mg/kg/week or less) is not considered sufficiently immunosuppressive to create vaccine safety concerns. Higher doses of methotrexate should be avoided.


200

Draw a basic synovial joint and then draw the formation of a pannus and how this would affect our patient. Include articular cartilage, synovial membrane, synovial fluid, and articular capsule.

:)

200

In the initial presentation of RA, these two specific factors are typically elevated.

With these factors in mind, draw which cells they come from and what other effects these factors influence in the immune response 

IL-1, TNF

300

A 38-year-old female visits your office complaining of several years of joint swelling and stiffness that is worse in the morning and improves throughout the day. Physical examination reveals bilateral deformities at her proximal interphalangeal and metacarpophalangeal joints. The presence of which of the following in this patient’s serum would most help 'rule in' a diagnosis of rheumatoid arthritis


A. Rheumatoid factor

B. Anti-nuclear antibody

C. Anti-citrullinated protein antibody

D. Anti-centromere antibody

E. Anti-smooth muscle antibody


Answer: C

The patient’s symptoms are indicative of rheumatoid arthritis (RA). Anti-citrullinated protein antibody (ACPA, or anti-CCP) has greater than 90% specificity for diagnosing RA.

300

A 5-year-old boy and his mother present to the pediatrician. The child has a 2-month history of intermittent fevers, joint pain, and tenderness in seven joints and a rash that occasionally develops across his buttocks. The rash is not present when the physician examines him. The child weighs 18.2 kg (40 lb). Which one of the following disorders does the patient likely have?

A. RA

B. Systemic JIA

C. Polyarticular JIA

D. Oligoarticular JIA

E. Osteoarthritis

Answer: B Systemic JIA

300

This is the significance of a prednisone taper. Be specific as to what would happen if someone didn't taper. 

What is "all of the nasty side effects of prednisone/corticosteroids"?

Adrenal suppression: May cause hypercortisolism or suppression of hypothalamic-pituitary-adrenal (HPA) axis, particularly in younger children or in patients receiving high doses for prolonged periods. HPA axis suppression may lead to adrenal crisis. Withdrawal and discontinuation of a corticosteroid should be done slowly and carefully. Adult patients receiving >20 mg per day of prednisone (or equivalent) may be most susceptible. 

Immunosuppression: Prolonged use of corticosteroids may increase the incidence of secondary infection, mask acute infection (including fungal infections), prolong or exacerbate viral infections, or limit response to killed or inactivated vaccines. Exposure to chickenpox or measles should be avoided. Corticosteroids should not be used to treat viral hepatitis or cerebral malaria. Close observation is required in patients with latent tuberculosis and/or TB reactivity; restrict use in active TB (only fulminating or disseminated TB in conjunction with antituberculosis treatment). Latent or active amebiasis should be ruled out in any patient with recent travel to tropic climates or unexplained diarrhea prior to corticosteroid initiation. Use with extreme caution in patients with Strongyloides infections; hyperinfection, dissemination and fatalities have occurred.

Osteoporosis: Use with caution in patients with or who are at risk for osteoporosis; high doses and/or long-term use of corticosteroids have been associated with increased bone loss and osteoporotic fractures.

300

Draw a schematic detailing the interactions between genetic & environmental factors that affect the disease process of RA

[see M.E.'s notes]

300

A 17-year-old boy is preparing for elective surgery. During his preoperative work-up, it is discovered that he has had recurrent lung infections, otitis media and sinusitis for most of his life. He is referred to a specialist for further evaluation. Laboratory studies show the following:

Ig isotype - Patient’s value (mg/dL) 

IgM – 160

IgG – 770

IgA – 4

 

Which of the following additional finding is most likely to be seen in this patient? 

A. Contact dermatitis.

B. Decreased phagocyte efficiency.

C. Immune complex disease.

D. Increased atopy.

E. Partial albinism.

The correct answer is D.

This patient has selective IgA deficiency. The serum values quoted in this case are normal for all isotypes except IgA, which is significantly depressed. 

Selective IgA deficiency is the most common immunodeficiency disease in the United States.

Patients have a tendency toward recurrent pulmonary and upper respiratory infections as well as atopic allergy.

Patients can also present with infectious diarrheas.

These patients seem to isotype switch to IgE more frequently because of their inability to produce IgA. It is important to note that many patients with selective IgA deficiency have anti-IgA IgE antibodies. As such, they may have severe allergic reactions to blood products. Interestingly, these patients can receive blood products safely from other IgA-deficient donors.


Why the other answers are wrong

Choice A: Contact dermatitis is a type of delayed-type hypersensitivity that depends on TH1 cells, macrophages, and cytotoxic T lymphocytes. It is not increased in patients with selective IgA deficiency.


Choice B: Decreased phagocytic efficiency is the hallmark of chronic granulomatous disease, but it is not often associated with changes in immunoglobulin.

Choice C: Immune complex disease is more common in patients with complement deficiency. C3b plays an important role in the clearing of immune complexes (antigen-antibody) from the serum. It would not be associated with selective IgA deficiency.

Choice E: Partial albinism is found in Chédiak-Higashi syndrome, which is a defect in phagolysosome fusion within phagocytic cells. IgA is normal.

400

A 42-year-old woman presents complaining of pain in her hands. She reports that the pain is in both hands, and that it is usually worse in the morning. She reports that her hands are also stiff in the morning, but that this gradually improves throughout the morning. She notes, however, that her symptoms seem to be getting worse over the last three months. What is the most likely pathogenesis of her disease process?


A. Repetitive microtrauma

B. Type 1 hypersensitivity reaction

C. Production of antibodies against smooth muscle

D. Production of antibodies against antibodies

E. Anti-neutrophil cytoplasmic antibody production

D-

The patient in this vignette has rheumatoid arthritis (RA). RA is related to B-cell production of rheumatoid factor, which is an antibody against the Fc portion of IgG. RA is a systemic, symmetric, inflammatory arthritis. It classically presents with pain and stiffness, particularly of the hands. The distal interphalangeal (DIP) is classically spared. Symptoms are often worse symptoms in the morning and improve with use. For examinations, it is useful to compare RA to osteoarthritis, which often gets worse throughout the day and often involves the DIP joints.

400

Case 1:

A 78-year-old woman comes to the physician because of pain and stiffness in her hands for six months. She has had stiffness in the morning that improved after 20 minutes. She occasionally takes ibuprofen when the pain is unbearable, but is generally reluctant to take medication. She has a history of psoriasis and hypertension. Current medications include a topical steroid and hydrochlorothiazide. Her blood pressure is 123/74 mm Hg. Examination is shown.

<img data-fr-image-pasted="true" data-imagetype="AttachmentByCid" data-custom="AQMkADk0OWNlMzc4LTBmMDgtNDQCLTkxZjUtMDNjOGU0MTViNThhAEYAAANHwy14K%2FLTR6IO4dn8sL4XBwB9w94VIjJYT6J5Nb5ErSuIAAACAQwAAAB9w94VIjJYT6J5Nb5ErSuIAAAA%2FH3AowAAAAESABAAVfAP%2FYYuHkOwP6iW1xbfEw%3D%3D" src="data:image/png;base64,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

The correct answer is B.

The patient described has typical symptoms of osteoarthritis, including morning stiffness that resolves, worsening pain with activity and no evidence of swelling or inflammation.

 

Why the other answers are wrong:

Choice A: Gout is a peripheral arthritis that results from the deposition of sodium urate crystals in one or more joints. There is usually a spontaneous onset of pain, edema, and inflammation (swelling and redness) with gout. The most common joint involved is the metatarsal-phalangeal joint of the great toe (podagra). Other than the great toe, other common sites of gouty arthritis are the ankle, wrist, and knee.

Choice C: Psoriatic arthritis is not the correct answer. The patient's past medical history for psoriasis does not mean she suffers from psoriatic arthritis. Only 5 percent of patients who have psoriasis develop psoriatic arthritis, which presents with swelling, redness, and more than 30 minutes of morning stiffness. Distal interphalangeal (DIP) joint involvement and nail pitting is classic.

Choice D: Reactive arthritis, formerly known as Reiter syndrome, usually presents with a history of GI or genitourinary infection, most commonly after Campylobacter jejuni or Chlamydia trachomatis. The classic triad is urethritis, conjunctivitis and arthritis.

Choice E: Rheumatoid arthritis is also an inflammatory arthritis associated with swelling and redness, morning stiffness lasting longer than 30 minutes, and joint pain that gets better with activity. It usually involves the proximal interphalangeal (PIP), metacarpophalangeal (MCP) and wrist joints. There may also be constitutional symptoms. An x-ray film may show bony erosions, cysts, osteopenia, joint space swelling, calcifications and narrowed joint space.

400

Which of the following biologic agents commonly used to treat rheumatoid arthritis (RA) DOES NOT target tumor necrosis factor-alpha (TNF-alpha)?


A Infliximab

B Rituximab

C Etanercept

D Golimumab

E Adalimumab

What is Rituximab

400

Outline the comorbidities associated with systemic glucocorticoids (pathogenesis schematic) (hint: start with MOA of drug) 

Examples of things on schematic:

Cushing’s syndrome, decreased serum potassium, diabetes mellitus, fluid retention, growth suppression (children), hypokalemic alkalosis, hypothyroidism (enhanced), menstrual disease, negative nitrogen balance (due to protein catabolism), sodium retention

Hypersensitivity: Anaphylaxis, hypersensitivity reaction

Infection: Infection

Neuromuscular & skeletal: Amyotrophy, aseptic necrosis of bones (femoral and humeral heads), osteoporosis, pathological fracture (long bones), rupture of tendon (particularly Achilles tendon), steroid myopathy, vertebral compression fracture

Ophthalmic: Exophthalmos, glaucoma, increased intraocular pressure, subcapsular posterior cataract

Miscellaneous: Wound healing impairment

400

Janus kinases may influence the pathogenesis of RA by:

A. Releasing cytokines

B. Releasing antibodies

C. Presenting antigens to B lymphocytes

D. Enhancing cell signaling

E. Binding CD28 receptors to antigen-presenting cells

Answer: D

Continuous activation of JAK/STAT signaling in RA synovial joints results in the elevated level of matrix metalloproteinase gene expression, increased frequency of apoptotic chondrocytes and most prominently ‘apoptosis resistance’ in the inflamed synovial tissue.

500

Describe the natural progression of RA and the associated timeline without treatment.

RA may begin slowly and insidiously with malaise, fatigue, and generalized musculoskeletal pain, likely mediated by IL-1 and TNF. After several weeks to months the joints become involved. The pattern of joint involvement varies, but it is generally symmetrical and the small joints are affected before the larger ones. Symptoms usually develop in the hands (metacarpophalangeal and proximal interphalangeal joints) and feet, followed by the wrists, ankles, elbows, and knees. Uncommonly the upper spine is involved, but the lumbosacral region and hips are usually spared.

Involved joints are swollen, warm, painful, and particularly stiff when rising in the morning or following inactivity. The typical patient has progressive joint enlargement, decreased range of motion evolving to complete ankylosis, with the greatest damage occurring in the first 4 or 5 years. Approximately 20% of affected individuals enjoy periods of partial or complete remission, but the symptoms inevitably return and involve previously unaffected joints. A minority of individuals (10%) have an acute onset over several days with severe symptoms and polyarticular involvement.

Inflammation in the tendons, ligaments, and occasionally the adjacent skeletal muscle frequently accompanies the arthritis and produces the characteristic radial deviation of the wrist, ulnar deviation of the fingers and flexion-hyperextension of the fingers (swan-neck deformity, boutonnière deformity). The end result is a joint that has no stability and minimal or no range of motion. Large synovial cysts, like the Baker cyst in the posterior knee, may develop as the increased intra-articular pressure causes herniation of the synovium. Radiographic hallmarks are joint effusions and juxta-articular osteopenia with erosions and narrowing of the joint space and loss of articular cartilage

Long-term complications include systemic amyloidosis ( Chapter 6 ) in 5% to 10% of patients and infection with opportunistic organisms in patients who receive long-term anti-TNF or other immunosuppressive agents.

500

A 38-year-old man comes to the office because of ankle pain that has been ongoing for the past few months. He reports difficulty walking. He also states that his left index finger and right wrist felt as if they were sprained. One morning, the pain in his joints was so intense that he could hardly get out of bed. His primary care physician ordered ANA, ACPA, and RF titers, all of which came back negative. Which of the following is the most likely diagnosis?

A - Gout

B - Polymytosis

C - Psoriatic Arthritis 

D - RA

E - SLE

Psoriatic arthritis is a type of seronegative spondyloarthropathies, associated with tissue-type HLA-B27. It classically involves the DIP joints of the hands and feet, and can lead to sausage fingers or toes.

Main explanation

Psoriatic arthritis is a type of inflammatory joint disorder in the group of seronegative spondyloarthropathies. It is characterized as seronegative, due to the absence of rheumatoid factor (RF). It is also a disorder that is more common in individuals with tissue-type HLA-B27. Psoriatic arthritis is seen in a group of patients with the chronic skin disorder, psoriasis. Psoriasis is characterized by scaly skin lesions, most commonly at extensor surfaces. It can also affect the nails, and cause pitting, oncholysis - or the separation of the nail from the nail bed. Psoriatic arthritis is characterized by the involvement of axial and peripheral joints, especially the distal interphalangeal (DIP) joints of the hands and feet. The prolonged inflammation can lead to the swelling of the fingers or toes, also known as dactylitis.

500

A 54-year-old man comes to the physician for a follow-up examination because of progressive rheumatoid arthritis unresponsive to high dose nonsteroidal anti-inflammatory drugs (NSAIDs). Treatment with prednisone and hydroxychloroquine was started six weeks earlier. The patient is concerned about steroid-induced osteoporosis, because his father, a type 2 diabetic, recently fell and broke his hip. Laboratory studies and a dual energy x-ray absorptiometry (DEXA) test of the spine and hip are ordered to address the patient's concerns.

Which of the following additional tests would be recommended for this patient?

A. 1,25-dihydroxy vitamin D levels

B. High-density lipoprotein levels

C. Parathyroid hormone (PTH) levels

D. Serum glucose levels

E. Serum protein electrophoresis

Answer: D

Long-term corticosteroid therapy in a patient with a family history of diabetes may increase the chance of a patient developing that disease. Glucocorticoids increase hepatic gluconeogenesis, inhibit glucose uptake, and induce insulin resistance

500

This is the pathway through which RA and Osteoporosis interact (include location, relevant cells, cytokines & their effects) 

What is: 

TNF-alpha induces T-cells in the synovial space to express RANK-L (receptor activator for nuclear factor κ B ligand). RANK-L binds to RANK on the surface of osteoclasts, increasing the process of bone resorption. An imbalance of bone building and bone resorption (more bone vs. more not-bone) is the key factor in development of osteoporosis. 

500

A 60-year-old woman presents with progressively decreasing vision. Of note, she states that she has difficulty driving at night and discerning traffic lights. She thinks that she can still read the newspaper without difficulty at home. She has a history of poorly controlled diabetes and rheumatoid arthritis controlled with prednisone and hydroxychloroquine. She has a 40-pack-year history of smoking. On fundoscopy, she has the findings as seen in Figure A and Figure B. Which of the following is the most likely cause of her symptoms?

(Marlen's Question) 

Figure A:

Figure B:

** Non-enzymatic glycation of proteins

** Glucocorticoids

** Tobacco use

** Hydroxychloroquine

** Female gender

A

The etiology that most encapsulates all of this patient's symptoms is hydroxychloroquine retinitis. While cataracts can cause most of her visual concerns, only retinal damage from the hydroxychloroquine explains color vision changes. Cataracts develop slowly and painlessly, gradually manifesting as decreased vision. Typical descriptions include difficulty reading in dim lighting and glare from lights while driving at night. Cataracts are formed through multiple methods such as stiffening of the central lens material, changes in the lens proteins, and pigmentation of the lens. Hydroxychloroquine has not been found to be associated with cataract development. Symptoms of retinopathy include central vision loss, difficulty reading, changes in color vision, and central blind spots. Patients taking hydroxychloroquine should be annually evaluated by an ophthalmologist.

M
e
n
u